Integral triple ∭R3d3q δ3(q⃗ )(p⃗ ⋅q⃗ )2q2∭R3d3q δ3(q→)(p→⋅q→)2q2\iiint_{\mathbb{R}^3} d^{3}q ~\delta^ {3}(\vec{q})\frac{(\vec{p}\cdot\vec{q})^2}{q^{2}} que implica la función Delta de Dirac

estoy tratando de encontrar

R 3 d 3 q   d 3 ( q ) ( pag q ) 2 q 2 ,
dónde pag es un vector fijo.

La respuesta debería ser pag 2 3 . A continuación se muestra mi intento, que parece conducir a la respuesta incorrecta pag 2 2 .

Intento: Vamos a alinear q z con pag , entonces medimos θ bien pag . Como no hay ϕ dependencia para poder escribir

d 3 ( q ) = d ( q ) d ( θ ) 2 π q 2 pecado ( θ ) .

por lo tanto tengo

pag 2 0 d q d ( q ) π π d θ d ( θ ) porque 2 θ .

Entiendo

0 d ( q ) d q = 1 2 ,
si tratamos d ( q ) como un caso límite de una distribución gaussiana simétrica. Mientras que la θ integral es 1 . Entonces mi respuesta a mi pregunta es pag 2 2 . Cuál es diferente de la respuesta correcta pag 2 3 .

Entonces mis preguntas son:

  1. ¿Qué salió mal en mi derivación?

  2. ¿Cómo derivas y justificas la respuesta? pag 2 3 de primeros principios?

@Kyle Kanos: dado que esta integral triple generalmente no se define en matemáticas, este podría ser un caso en el que se requiere la perspectiva de un físico, cf. meta.física.stackexchange.com/q/5713/2451
¿Por qué complicar esto con coordenadas cilíndricas? solo use coordenadas cartesianas, y deje d 3 ( pag ) = d ( pag X ) d ( pag y ) d ( pag z )
Por definición, la distribución delta d ( 3 ) ( q ) devuelve el valor de la función integrada en q = 0 . A menos que pag = 0 , la función | pag q | 2 / q 2 no tiene valor en 0 . Por lo tanto, el delta no puede operar. Compruebe cómo se obtuvo esta receta integral. Probablemente haya un error en alguna parte.
@Jerry: ya lo intenté, no simplifica nada.
@Jan No creo que haya un error, este dirac delta vino como resultado de la integración mi i q . X en general X espacio. mi predicción de pag 2 3 es por el hecho de que obtengo LHS = RHS en mis ecuaciones.
Si quieres, publica el procedimiento que conduce a esta integral. Tal vez entonces podamos resolver este problema.
Evaluando la integral triple como tres integrales sucesivas en coordenadas cartesianas, q i , i { 1 , 2 , 3 } , da el resultado pag i 2 , Si el q i coordenada es la integración más externa. (sin suma sobre i .) Entonces moralmente, en promedio, eso da el resultado pag 2 3 .

Respuestas (2)

Sugerencias:

  1. En matemáticas, una distribución generalmente solo se define wrt. funciones de prueba suaves. Sin embargo la función q ( q pag ) 2 / q 2 no es continua en el origen q = 0 . No obstante, podemos, por ejemplo, intentar evaluar la integral triple usando la siguiente representación de la distribución delta 3D de Dirac

    (1) d 3 ( q )   =   límite ε 0 + 1 4 π 3 ε ( q 2 + ε ) 5 2 , q   :=   | q | ,
    donde se entiende implícitamente que el límite límite ε 0 + debe tomarse después de la triple integración.

  2. por dado ε > 0 , el integrando es integrable en R 3 . y está acotado en el origen q = 0 , por lo que podemos usar coordenadas esféricas. Como menciona OP, en coordenadas esféricas con pag a lo largo de z -eje, tenemos

    (2) ( q pag ) 2 q 2   =   pag 2 porque 2 θ .

  3. Sustituto q ε q en la integral triple. El ε -desaparece la dependencia. Realiza la integral triple.

¡Gracias! Pero me gustaría preguntar por qué su definición de delta de Dirac es independiente de los ángulos, ya que tengo un porque 2 θ en el integrando?

d 3 ( q ) = d ( q ) d ( θ ) 2 π q 2 pecado ( θ )
Está Mal. La función delta es esféricamente simétrica y, por lo tanto, no tiene dependencia de θ. Simplemente use:
d 3 ( q ) = d ( q ) 2 π q 2
en cambio. Utilice el jacobiano cuando cambie de sistema de coordenadas (de cartesiano a esférico) ( r 2 pecado ( θ ) ), y debería obtener el resultado.

¡Gracias! Sin embargo, estoy confundido acerca de la porque 2 θ término, pensé que eso lo hacía no esféricamente simétrico. ¿Qué quiere decir con que el delta de dirac es esféricamente simétrico, no es un delta de dirac, por definición, siempre esféricamente simétrico?
Es por definición esféricamente simétrica. el cos( t h mi t a ) el término proviene del producto escalar que multiplica la función delta, no de la función delta en sí...
Estoy de acuerdo, pero no es la forma más general de d ( r ) = d ( r ) d ( θ ) d ( ϕ ) r 2 pecado θ en coordenadas esféricas? ¿No es el θ del producto escalar relacionado con este θ ?